PT23.S3.Q14 - if the proposed tax reduction

LsatbreakingnewsLsatbreakingnews Alum Member
edited August 2017 in Logical Reasoning 392 karma

Hi guys. I am having trouble with an LR question.
PT 23 S3 Q14

Premises: If tax adopted -> Discontinue story hours -> parents inconvenienced

Conclusion: Tax reduction package not adopted

So in my mind to get to this we would have to know either that parents are not inconvenienced or story hours are not discontinued

so: ~(Parents incon) -> ~(discontinue story) -> ~(tax adopted)

Maybe I am not translating correctly, but I don't see how the correct answer D leads to this.
https://7sage.com/lsat_explanations/lsat-23-section-3-question-14/

Comments

  • LSATcantwinLSATcantwin Alum Member Sage
    edited August 2017 13286 karma

    Okay well let's see what is going on.

    So the stimulus tells us;

    1.) If the tax reduction package is adopted, then the library will have to stop story time.

    2.) If story time is stopped, then many parents will be inconvenienced.

    So (conclusion indicator)

    Conclusion - The tax reduction package will not be adopted this year.

    Okay so lets link this out,

    TPA -> LSST -> PI


    /TPA

    Well we're missing something here. From what we are given so far we can not properly conclude that the Tax Reduction Package will not be adopted this year.

    Let's take a look at answer choice D.

    D.) No tax reduction package that would greatly inconvenience parents will be adopted this year.

    So what does that mean?

    Well it means that the tax reduction package that we are talking about, that would inconvenience parents, will not be adopted this year.

    It basically says;

    If a tax reduction plan inconveniences parents, then it won't be adopted this year.

    We know that this tax reduction plan will cause the library to stop story time. This stopped story time will greatly inconvenience parents. So it will not be adopted this year.

    Now our conclusion follows.

    Does this help at all?

  • sjiang666sjiang666 Alum Member
    edited August 2017 157 karma

    I think your translation is fine. What's in your mind is a possible but not the only way to justify the argument. You need to be open to all possible ways that could link P and C. Why D is right? because it means:

    If a tax reduction plan inconveniences parents, then it won't be adopted this year.

    According to your idea, the right answer will looks like "the parents can not be inconvenienced." While it justifies the argument, it doesn't sound as good as D, right? And this will also make the right answer too awkward and obvious.

Sign In or Register to comment.